LSAT and Law School Admissions Forum

Get expert LSAT preparation and law school admissions advice from PowerScore Test Preparation.

 Administrator
PowerScore Staff
  • PowerScore Staff
  • Posts: 8916
  • Joined: Feb 02, 2011
|
#59037
Please post your questions below!
 More_Monitor
  • Posts: 1
  • Joined: Aug 13, 2020
|
#78047
I am not understanding the distinction between answer choice A and answer choice D.
This is how I have interpreted the stimulus:
The argument states that the promotion was counterproductive because last year we had 640 visits/rep and this year we had 501 visits/rep. You need to consider whether there is a different reason why there are less visits per rep. For example, the number of visits per rep may be lower but that does not mean that there is less willingness to receive sales rep visits, but rather there's more people splitting the work so reps have to visit less physicians.

A) good for explaining an alternative reason as to why we have less visits per reps and I will keep this.

B,C,E - immediately eliminated

D) I selected this because in my mind this also could explain why there are less visits per sale reps because they could be visiting the same number of physicians but just less frequently.
 Paul Marsh
PowerScore Staff
  • PowerScore Staff
  • Posts: 290
  • Joined: Oct 15, 2019
|
#78132
Hi More_Monitor! Your interpretation of the stimulus sounds exactly right. My Pre-Phrase for this question was very similar to yours - "maybe there's a different cause, such as more representatives splitting the work!".

(A) matches that Pre-Phrase almost exactly, so we have to feel good about that one.

As for (D). In order for (D) to do what you're saying it does, we have to make multiple assumptions. We have to assume that two years ago more representatives were doing repeat visits to physicians, and then we have to assume that last year fewer representatives were doing repeat visits to physicians (nowhere is this decrease in multiple visits suggested by (D)). Even if those assumptions were true, I'm not sure (D) actually weakens the argument. Remember that the conclusion of the argument is that physicians were less inclined last year to receive visits from representatives. If physicians were receiving fewer multiple visits from representatives last year, that sort of seems to strengthen the conclusion that physicians didn't want to see them as much! So a couple things here: 1) if you have to make assumptions about your answer choice in order to have it Weaken the argument then it's likely wrong, and 2) make sure your answer actually Weakens the argument!

Hope that helps!
 g_lawyered
  • Posts: 211
  • Joined: Sep 14, 2020
|
#97640
Hi PS,
When weakening this casual argument, can't we weaken it by weakening the conclusion directly? I prephrased that argument can be weaken if it introduced a way why the doctors would want to receive pharm. reps visits (the opposite of what the conclusion is saying). With this prephrase, I chose (B) because the doctors receiving free samples from pharm. reps would lead to doctors wanting visits from them (because the doctors receive benefit from pharm. rep visits). Why is this incorrect?

Is (C) incorrect because it strengthens the conclusion by eliminating an alternate explanation. Pharm. companies not spending the extra money on consumers, confirms that the extra money was spent on pharm. reps to visit doctors. This confirms the conclusion that the "additional promotion (spending extra money) was the reason doctors don't want pharm. rep visits. This was my reason for eliminate (C). Is this why (C) is incorrect?

Also, is (D) incorrect because it strengthens the conclusion that because pharm. reps are visiting the same doctors more than once (meaning the pharm. reps visit less doctors) strengthens that the amount of doctors are less (than previous year). Less doctors are receiving pharm. reps visits. This was my reason for eliminate (D). Is this why (D) is incorrect?

Please help my strategy on Weaken questions :-?
Thanks in advance!
 Robert Carroll
PowerScore Staff
  • PowerScore Staff
  • Posts: 1787
  • Joined: Dec 06, 2013
|
#97804
g_lawyered,

We are definitely allowed to weaken the conclusion directly. Answer choice (B) doesn't match your prephrase. It's saying that doctors who DO receive rep visits get something else as well. But the entire point of the stimulus is that it appears that doctors are receiving fewer rep visits anyway. Talking about the remaining few doctors who DO receive reps isn't weakening the argument, nor is it affecting it the way you want from your prephrase. You're taking answer choice (B) to be talking about all doctors. It's not - it's explicitly limited to those who continue to receive visits. It's therefore prima facie about a more limited group of doctors, and would do nothing to doctors who aren't accepting reps anymore. The question that remains is...are there a lot of doctors not accepting reps, like the author thinks? Answer choice (B) doesn't help OR hurt that.

I wouldn't say answer choice (C) strengthens. I think it does nothing, but to the extent that your explanation means it would strengthen if it even were relevant, that seems to kill the answer. If it were even relevant, it would only be to strengthen the argument.

Since the number of total visits per rep has gone done, if reps were always making multiple visits, you're right that answer choice (D) strengthens the argument.

Robert Carroll
 g_lawyered
  • Posts: 211
  • Joined: Sep 14, 2020
|
#97812
Hi Robert,
I completely missed that important point about the 2 groups of doctors mentioned in the argument as you pointed out. That's more clear to me now. Thanks for your help!

Get the most out of your LSAT Prep Plus subscription.

Analyze and track your performance with our Testing and Analytics Package.